Find the slope……….—-

Find The Slope.-

Answers

Answer 1

[tex](\stackrel{x_1}{0}~,~\stackrel{y_1}{3})\qquad (\stackrel{x_2}{5}~,~\stackrel{y_2}{-1}) ~\hfill \stackrel{slope}{m}\implies \cfrac{\stackrel{rise} {\stackrel{y_2}{-1}-\stackrel{y1}{3}}}{\underset{run} {\underset{x_2}{5}-\underset{x_1}{0}}} \implies \cfrac{ -4 }{ 5 } \implies - \cfrac{ 4 }{ 5 }[/tex]


Related Questions

Determine the x-coordinate that would yield a y-coordinate of 30, assuming the pair follows the same trend that currently exists in the data set below. {(45, 18), (38, 21), (26, 31), (64, 13), (29, 28), (33, 25), (49, 17)}

Answers

Based on the trend found in the data set, the x-coordinate that would yield a y-coordinate of 30 is 28

What is the trend in the data set?

Pick two points from the data set:

(26, 31) and (64, 13)

The slope is:

= Change in y / Change in x

= ( 13 - 31) / (64 - 26)

= -18 / 38

= -9/19

The y-intercept is:

y = mx + b

13 = -9/19(64) + b

b = 43.3

The x-coordinate that gives a y-coordinate of 30 is:

30 = -9/19x + 43.3

x = 28

Find out more on trend in data set at https://brainly.com/question/24679525

#SPJ1

What is the opposite of 6?

Answers

Answer:

1

Step-by-step explanation:

The opposite of a number is the number on the other side of 0 number line, and the same distance from 0.

Johanna bought 17 items at the college bookstore. The items cost a total of ​$41.50. The pens cost $0.50 ​each, the notebooks were ​$4.00 ​each, and the highlighters cost ​$1.50 each. She bought 3 more notebooks than highlighters. How many of each item did she​ buy?

Answers

The answer to the given word problem is as follows. From the calculations: Johanna purchased

4 pens, 8 notebooks, and 5 highlighters.What is a word problem?

A word problem is a few phrases that describe a real-life scenario in which an issue must be solved using a mathematical computation.

The calculations is given as follows:

Step 1 -  First, let's define the variables to represent the unknowns.

Let p = number of pens

Let n = number of notebooks

Let h = number of highlighters

p + n + h = 17 ......................... 1  (purchased 17 items)

.5p + 4n + 1.5h = 41.5 ...................2 (total cost $41.50)

n = h+3  ................................3 (3 more notebooks than highlighters)

Step 2...............Solve for P in equation 1

P = 17 - n - h..................4

Recall that in equation 3,

n = h + 2

If we make n the subject of the expression, we have:

n = h+ 3

Substituting that into equation 4, we have

p = 17 - (h+3) - h
p = 17 -2h - 3 ..................5

Going back to equation 2, let's substitute n with h+3 and replace p with 17 -2h - 3

.5(17 -2h - 3) + 4 (h+3) + 1.5h = 41.5

To remove all decimals, we can multiply all with 10 to get:

5(170-20h -30) + 40(10h +30) + 15h = 415

Expand all brackets and we have:

850 - 100h - 150 + 400h + 1200 + 15h = 415

Collect like terms

-100h + 400h + 15h = 415-850+150-1200

315h = -1485

h = -1485/315

h = -4.71

Since a physical item cannot be negative, we utilize the absolute value which is:

h = 4.71

h [tex]\approx[/tex] 5 highlighter

Recall that

n = h + 3

Hence

n = 5 +3 = 8 notebooks

Recall that

p + n + h = 17

Hence we have

p + 8 + 5 = 17

p = 17 -8 - 5

p = 17 - 13
P = 4 pens.

Hence Johanna purchased

4 pens, 8 notebooks and 5 highlighters.

Learn more about Word Problems:
https://brainly.com/question/13818690
#SPJ1

I need help fast please. I also need explanation on an easy way to do it.

Answers

The correct equation that is balanced using the given numbers once is;

5(-1 + (-3)) = (7 * -4) + 8

How to carry out Algebra Properties?

There are different properties of Algebra such as;

Commutative Property of Addition or multiplication

Associative Property of Addition or multiplication

Distributive Property

Now, from the given numbers, it will be best to use only a positive digit outside the bracket on the left hand side to make it easy to balance on the right hand side. Thus, for the left hand side we will try;

5(-1 + (-3))

When that is solved we will get -20

Now, to balance that on the right hand side , we will have;

(7 * -4) + 8

Read more about Algebra Properties at; https://brainly.com/question/11131460

#SPJ1

A certain illness is spreading
at a rate of 10% per hour.
How long will it take to
spread to 1,200 people if 3
people are initially exposed?
Round to the nearest hour.

Answers

The illness is spreading at a rate of 10% per hour -> it is spreading 1.1x faster per hour as 10% more people are infected each time.

Therefore, we can write the amount of people infected as 3 x [tex]1.1^{n}[/tex], where n is the hour. So, if n equals 1, the amount of people infected is 3 x 1.1 = 3.3 (so when this mark surpasses the 1200 limit, we round up to the nearest whole number)

It will take 1200 : 3 : 1.1 = 363.636364.. (hours)

If it's only 363 hours, the number of people infected will not have reached 1200, as we have .636364 hours left. So, it will take 364 hours.

Please look at photo for further clarification, the length of a conduit is needed for sections and be shown in the picture. you may round the final answer to the nearest hundred if needed as well as all the intermediate values rounded to the nearest hundred if needed.

Answers

Using trigonometric functions :

From the problem, we have a hypotenuse of 22.25 cm and an angle of 31°28'

To find A, we will use the sine function :

[tex]\begin{gathered} \sin 31^{\circ}28^{\prime}=\frac{A}{22.25} \\ A=22.25\sin 31^{\circ}28^{\prime} \\ A=11.615 \end{gathered}[/tex]

To find B, we will use the cosine function :

[tex]\begin{gathered} \cos 31^{\circ}28^{\prime}=\frac{B}{22.25} \\ B=22.25\cos 31^{\circ}28^{\prime} \\ B=18.978 \end{gathered}[/tex]

The total length is the sum of A and B :

A + B = 11.615 + 18.978 = 30.59

The answer is 30.59 cm

Need help on this geometry problem, solving for x.

Answers

Answer:

A.) 30 degrees

Step-by-step explanation:

Brainly pls :)

Graph a line with a slope of 1/4 that contains the point (6,3) khan academy

Answers

4y = x + 6 is the equation of the line using the slope value.

What is a line's slope?

The steepness of a line segment is explained by the slope of the line. It is a ratio of the vertical x-axis coordinates and the y-axis coordinates. The classification of whether two lines are parallel or perpendicular depends on the slope value.

The parameters for the line segment are as follows, in accordance with the question:

The coordinate points (6, 3) and the slope of a line (1/4) are as follows: (x = 6; y = 3)

Using the line segment's standard equation now: y = mx + c

If (x, y) are coordinates, "m" is the slope, "c" is the y-intercept,

The following results are obtained by replacing the slope and y-intercept values in the formula:

y = mx + c

⇒ 3 = (1/4)(6) + c

⇒ c = 3 - 3/2 = 3/2

Consequently, c = (3/2) is the y-value. intercept's

by changing the slope value and y-intercept value in the line's equation:

y = (1/4)x + (3/2) = 4y = x +6

Consequently, 4y = x + 6 is the equation of the line using the slope value.

To learn more about the slope of a line from the given link:

https://brainly.com/question/16949303

#SPJ10

Hey there! I am having trouble on a quiz so I'll be asking some questions. Look at my profile if you are looking to earn points and help someone out! ( 25 Points For Each Question Answered Correctly - Do Not Comment Silly Stuff Or Your Points Will Be Removed.)

1st Question:

Answers

The simplified form of the expression 4(4y-[tex]7y^{2}[/tex])-9(5y+2) is -28[tex]y^{2}[/tex]-29y-18

The given the expression is 4(4y-[tex]7y^{2}[/tex])-9(5y+2)

To solve this problem first we have to use the distributive property

The distributive property states that multiplying the sum of two or more variables by a number gives the same result as when each variable is multiplied individually by the number and the products are added together

The distributive property of addition

A(B+C) = AB + AC

The distributive property of subtraction

A(B-C) = AB - AC

The given equation is 4(4y-[tex]7y^{2}[/tex])-9(5y+2)

Apply distributive property

4(4y-[tex]7y^{2}[/tex])-9(5y+2) = 16y -28[tex]y^{2}[/tex]-45y-18

Rearrange the terms

-28[tex]y^{2}[/tex]-45y+16y-18 = -28[tex]y^{2}[/tex]-29y-18

Hence, the simplified form of the expression 4(4y-[tex]7y^{2}[/tex])-9(5y+2) is -28[tex]y^{2}[/tex]-29y-18

Learn more about distributive property here

brainly.com/question/13130806

#SPJ1

If a space rover has a mass of 3900 kg, then what is its weight on Earth?
N=?
Use g-9.81m/s? and do not include units in your answer.

Answers

If a space rover has a mass of 3900 kg, then its weight on Earth N will be 38220 N.

What is a weight?

Weight can be described as the body's relative mass  which can be regarded as the quantity of matter that can be found in the particles of the object and the unit can be written as Newton.

To calculate the weight of a particular particles, we can use the formular (mg) which is that product of the mass of the object to that of the gravity.

from the question, the gravity is been given as  g-9.81m/s

Then the weight = (3900 kg* 9.8) = 38220 N

Learn more about mass  at:

https://brainly.com/question/25959744

#SPJ1

find the value of x please

Answers

when running a line, in a right-triangle, from the 90° angle perpendicular to its opposite side, we will end up with three similar triangles, one Small, one Medium and a containing Large one.  Check the picture below.

[tex]\cfrac{x}{7}~~ = ~~\cfrac{10}{x}\implies x^2=70\implies x=\sqrt{70}[/tex]

Making a equation with the slope: 2/3 and the y intersept:2

Answers

Answer:

Equation of line is given as y = mx + c, where m is the slope and c is the y-intercept.

Equation of line is y = 2/3x + 2

help pass due asap!!!
Add (7x³ - 2x² - 5x+6) and (2x³ + 3x² − x − 1)
a) 9x³ + x² - 6x +5
b) 9x6 + x4 + 5x² − 6
c) 9x³x² - 6x + 5
d) 14x³ - 6x² – 5x - 6

Answers

The answer to your problem is A

Fill in the missing number.
20% of
= 3

Answers

Answer:0.6

Step-by-step explanation:

20/3 is 6

Name a plane parallel to plane ABCD.
A. plane ADHE
B. plane DCGH
C. plane ABFE
D. plane EFGH

Answers

Answer:

Plane EFGH

Step-by-step explanation:

we can see that plane ABCD is on the bottom

and that plane EFGH is on the top

so if the go left and right, they will never touch that is what parallel is

any other plane will intersect or touch eventually

y = 3x + 2 is reflected in the line y=-1. What is the equation of the image?​

Answers

The equation of the image is y = 3x - 4

In this question, we have been given a line y = 3x + 2 is reflected in the line y = -1.

We need to find the equation of the image.

The reflected line must be parallel to the line y = 3x + 2

So, the slope of the image must be equal to the slope of the line y = 3x + 2

The y-intercept of the line y = 3x + 2 is (0, 2)

The distance between point (0, 2) and the line y = -1 is 3 units.

So, the y-intercept of the reflected line must be (0, -4)

With slope m = 3 and y-intercept (0, -4) the equation of the line would be,

y = 3x - 4

Therefore, the equation of the image is y = 3x - 4

Learn more about equation of line here:

https://brainly.com/question/24524587

#SPJ1


Ivanna makes 9 dollars for each hour of work. Write an equation to represent her total pay p after working h hours.

Answers

Answer:

9h=p

9 times the number of hours worked equals the total pay.

Adriana mows lawns. She charges an initial fee and a constant fee for each hour of work.

Answers

Answer: It's $12 an hour.

Step-by-step explanation:

HELP ASAP NEEDS TO BE DONE BEFORE TONIGHT

Answers

Answer:

y=x/4

Step-by-step explanation:

if the line is perpendicular (m2)=-1/(m1)

m2=1/4

y=x/4+c

sub in 8,2

2=8/4+c

c=0

equation is therefore y=x/4

The perimeter of a rectangular parking lot is 370 m.
If the width of the parking lot is 86 m, what is its length?

Answers

The length would be 4.302

Explanation: 370 divided by 86 is 4.30232 then just round

Answer:

the length is 99m

Step-by-step explanation:

you have to add 86 to 86 to get 172, subtract 172 from 370 then you have 198 then divide to have  99m the length.

Please put all the numbers above your phone on the number line

Answers

1.

(a) 2π - 9

= 2 x 3.142 - 9 = 6.284 - 9

= -2.716

This will be closest to the letter C on the number line.

(b) [tex]\sqrt{3}[/tex]

= 1.732

This will be closest to the letter T on the number line.

(c) [tex]\sqrt[3]{9}[/tex] - 8

= 3-8

= -5

This will be closest to the letter S on the number line.

What are Real numbers?In the number system, real numbers are only the fusion of rational and irrational numbers. These numbers can generally be used for all arithmetic operations and can also be expressed on a number line. Imaginary numbers, which are often known as unreal numbers since they cannot be stated on a number line, are frequently used to symbolize complex numbers. Real numbers include things like 23, -12, 6.99, 5/2, and so on.Most physical constants, like the universal gravitational constant, and physical variables, including position, mass, speed, and electric charge, are represented using real numbers in the physical sciences.Although the Zermelo-Fraenkel axiomatization of set theory is the method most frequently used to formalize the real numbers, other mathematicians also explore the real numbers using other logical mathematical foundations. Electronic calculators and computers, with a few notable exceptions, do not work with real numbers. Instead, they frequently use floating-point numbers, which are finite-precision approximations that resemble scientific notation.

To learn more about Real numbers, refer to:

https://brainly.com/question/551408

#SPJ13

write a algebraic expression 4 less than the sum of a number and 7

Answers

The representation of the expression is 4 < x + 7

How to translate the algebraic expression?

The mathematical statement is given as

"4 less than the sum of a number and 7"

Sum means Plus

So, we have the following representation

"4 less than the sum of a number and 7" ⇒  4 less than a number + 7"

Let the number be x

So, we have

"4 less than the sum of a number and 7" ⇒  4 less than x + 7"

Less than as used here means <

So, we have

"4 less than the sum of a number and 7" ⇒  4 < x + 7"

Hence, the expression represented by "4 less than the sum of a number and 7" is  4 < x + 7

Read more about expressions at

https://brainly.com/question/4344214

#SPJ1

3. Find the number of volunteers in the group if each volunteer cleans up a section of the following lengths. Be prepared to explain or show your reasoning.
a. 0.4 mile
b. 2/7 mile
c. 0.125 mile
d. 6/45 mile

Answers

The number of volunteers needed for each case is given as follows:

a. 0.4 miles: 5.

b. 2/7 miles: 7.

c. 0.125 miles: 16.

d. 6/45 miles: 15.

What is a proportion?

A proportion is a fraction of a total amount, and equations can be built to find the desired measures in the problem using the unit rates of the context of the problem along with basic arithmetic operations, especially multiplication and division.

In this problem, a two mile section will be cleaned, hence the number of volunteers needed is calculated as follows:

n = 2/s.

In which s is the section cleaned by each volunteer.

In item a, the section is of 0.4 miles, hence the number of volunteers is given as follows:

n = 2/0.4 = 5 volunteers.

In item b, the section is of 2/7 miles, hence the number of volunteers is given as follows:

n = 2/(2/7) = 2 x 7/2 = 7 volunteers.

In item c, the section is of 0.125 miles, hence the number of volunteers is given as follows:

n = 2/0.125 = 2/(1/8) = 2 x 8 = 16 volunteers.

In item d, the section is of 6/45 miles, hence the number of volunteers is given as follows:

n = 2/(6/45) = 2 x 45/6 = 90/6 = 15 volunteers.

More can be learned about proportions at https://brainly.com/question/24372153

#SPJ1

Rewrite 0.00518 in scientific notation.

Answers

Answer: 5.18 x 10^-3

Step-by-step explanation:

0.00518

**Move the decimal 3 spaces to the right**

5.18

**Since it goes like 0.00, you want to make sure the exponent is negative**

5.18 x 10^-3

Hope this helped <3

is each pair of lines parallel, perpendicular, or neither?

Answers

Answer:

1. perpendicular

2. parallel

3. neither

Step-by-step explanation:

The first set of lines are perpendicular because they intersect each other at a right angle.

The second set of lines are parallel because they never cross.

The first pair forms a 90° angle therefore the lines are perpendicular.

The second lines are straight that it's obviously clear they will never meet meaning they are parallel

the third lines are neither perpendicular nor parallel.

A lawnmower with a 3-gallon gas tank uses an average of 0.6 gallons per hour.

A function modeling this situation, where x represents the amount of gas in the tank, has a domain described by which inequality?
A. 0 ≤ x ≤ 5
B. 0 < x < 5
C. 0 < x < 3
D. 0 ≤ x ≤ 3

Answers

The inequality that describes the domain is 0 ≤ x ≤ 5

How to determine the domain of the situation?

The given parameters are

Size of tank = 3 gallons

Rate = 0.6 gallons per hour

The highest number of time is calculated as

Time = Size of tank/Rate

Substitute the known values in the above equation

So, we have the following equation

Time = 3/0.6

Evaluate

Time = 5

The domain is then represented as

0  ≤ x ≤ Time

This gives

0 ≤ x ≤ 5

Hence, the domain of the situation is 0  ≤ x ≤ 5

Read more about domain at

https://brainly.com/question/2264373

#SPJ1

the mean is 115 the other numbers that make the mean are 110 and 90 i cant figure out what the third number is

Answers

Answer: 145

Step-by-step explanation:

To find the total of the three numbers: 115 x 3 = 345

So the 3rd number must be 345 - 110 -90 = 145

Check the answer= 110 + 90 +145

                                         3

= 115

Mmkkknnnnnfjdjjddkbshaksisksisi

A case of 24 pairs of the same kind of switches cost a little more than $800 explain whether $28 per pair with tax included is a good estimate of the price

Answers

If a case of 24 pairs of the same kind of switches cost a little more than $800. $28 per pair with tax included is not  good estimate of the price  but  $33.33 is the good estimate for the price .

Estimated price

Assuming that the  cost per pair is the amount of $28. Hence, cost of 24 pairs is calculated as:

Cost= $28x24

Cost  = $672.

Based on the above calculation for the cost of 25 pairs we can see that that the  24 pairs of the same kind of switches does not cost more than $800 which implies that $28 is not good estimate for the price.

Now let determine the best estimate of the price with tax included :

Cost = $800/24

Cost = $33.33

Therefore the good estimate for the price with the tax included is $33.33.

Learn more about estimated price here: https://brainly.com/question/11292014

#SPJ1

HELP
25 POINTS!!!!!!!!!!!!!!!!!!!!!!11

Answers

Answer:

8/9 cups is the correct answer

Solve the following inequality. 3x - 4 > 11
A. (∞ ,5)
B. (15, ∞)
C. (5, ∞)
D. [5, ∞)

Answers

3x - 4 > 11


15 it’s not that hard
answer: D. (5, ♾)

explanation:

first u need to solve for x:

3x - 4 > 11

3x > 15

x > 5

so if x is greater than 5, then the domain is above 5 to infinity, written as (5, ♾) instead of [5, ♾) because the bracket means greater than or equal to, but > is just greater than

Other Questions
Clancy has 423 cups of yogurt to share with his friends at a party. Each guest will receive 1 serving of yogurt which is 34 of a cup.Write a multiplication equation to determine the number of people, p, who can be invited to the party. What is one reason that commonly confused word pairs can be difficult to find when editing? question 3 options: they usually use different verb tenses. they cannot be found in a thesaurus. they are spelled correctly so they are not found by spell-checkers. they cannot be found in a dictionary. whats the difference between narrative writing and procedural writing to calculate direct materials on the schedule of cost of goods manufactured, add purchases to beginning raw materials inventory and subtract ______. Which of the following are the coordinates of point F' , the image of point F, after a rotation of 90 about (1 point)the origin? Use the Chain Rule to find z/s and z/t. (Enter your answer only in terms of s and t. Please use * for multiplication between all factors.)[tex]z = x^{7}y^{7}, x = s cos(t), y = s sin(t)[/tex] Write whether each noun is a colective noun or abstract noun. 1. Team: 2. Hate: 3Vegetable . 4. Wealth: 5. Fear: This list of accomplishments can be attributed to which group?O SocialistsO MuckrakersO ProgressivesO Know-nothings When I want to get a package delivered to an access point do I put my address or the access point When improvising a new melody, you should use non-harmony notes on the strong beats of the bar and join them together with notes of the triad.A. TrueB. False, you should use the notes of the triad on the weak beats of the bar.C. False, you should use notes of the triad on the strong beats of the bar and join them together with non-harmony notes. Please help with number 8 step by step El volumen de un cubo que tiene como arista 14 cm es?1000 cm32.744 cm3216 cm3 in a herd of dairy cows the narrow-sense heritability for milk protein content in 0.76 and for milk butterfat is 0.82. the correlation coefficient between milk protein content and butterfat in 0.91. if the farmer selets for cows producing more butterfat in their milk, what will be the most likelyeffect on milk protein content in the next generation? Though these myths are very different on the surface, they contain many similar elements that cut across cultures and societies. in an essay of 300 words or more, identify one or two elements that are present in several of the creation myths presented, then write an analysis on why these elements may be universally important. cite specific evidence from the text to support your analysis. if a baseball player has a batting average of 0.330, what is the probability that the player will get the following number of hits in the next four times at bat? scalene triangles are symmetric. true or false Help please it's true or false The fuel tank of a certain airplane can hold up to 200 gallons of fuel. Let W be the total weight of the airplane (in pounds). Let F be the total amount of fuel inIts tank (in gallons). Suppose that W=5F+4000 gives W as a function of F.Identify the correct description of the values in both the domain and range of the function. Then, for each, choose the most appropriate set of values.Set of Values(Choose one)Domain:Range:Description of ValuesOweight of airplane (in pounds)Oamount of fuel in airplane's tank (in gallons)Oweight of airplane (in pounds)amount of fuel in airplane's tank (in gallons)(Choose one)X28180 at a certain point on a heated metal plate, the greatest rate of temperature increase, 6 degrees celsius per meter, is toward the northeast. if an object at this point moves directly north, at what rate is the temperature increasing? can someone help me with Spanish work